1. Trang chủ
  2. » Luận Văn - Báo Cáo

giải tích hàm nhiều biến yếu tố giải tích trong bất đẳng thức phương pháp nhân tử lagrange

19 0 0
Tài liệu đã được kiểm tra trùng lặp

Đang tải... (xem toàn văn)

Tài liệu hạn chế xem trước, để xem đầy đủ mời bạn chọn Tải xuống

THÔNG TIN TÀI LIỆU

Nội dung

Trang 1

TRƯỜNG ĐẠI HỌC SƯ PHẠM THÀNH PHỐ HỒ CHÍ MINH

KHOA TOÁN - TIN HỌC

GIẢI TÍCH HÀM NHIỀU BIẾNYẾU TỐ GIẢI TÍCH TRONG BẤT ĐẲNG THỨC

PHƯƠNG PHÁP NHÂN TỬ LAGRANGE

NHÓM THỰC HIỆN : NHÓM 1

GIẢNG VIÊN HƯỚNG DẪN: PGS.TS NGUYỄN THÀNH NHÂN

THÀNH PHỐ HỒ CHÍ MINH, 04-2022

Trang 3

GIẢI TÍCH HÀM NHIỀU BIẾNBÁO CÁO

Trang 4

II PHẦN BÁO CÁO

1 Đặt vấn đề

Ưu thế khi giải bất đẳng thức bằng các công cụ giải tích

Bất đẳng thức là một chuyên đề không mấy gần gũi với học sinh trung học bởi vì các emchß được tiếp cận khi ôn thi tuyển sinh lớp 10, hoặc khi là thành viên các đội tuyển đểtham gia các kỳ thi Toán, được tiếp cận với một vài câu trong đề thi THPTQG Tuy là mộtchuyên đề khó nhưng trong chương trình THPT học sinh chß được giới thiệu hai bất đẳngthức kinh điển là bất đẳng thức Cauchy và bất đẳng thức Bunhiacopxki trong sách giáokhoa Toán 10 Trong khi đó, bất đẳng thức đòi hỏi học sinh phải có tư duy nhạy bén, bảnthân đã được trang bị kinh nghiệm về các kết quả bổ trợ và cần nhiều thời gian cho việcluyện tập Đó cũng chính là lý do tập thể người viết muốn bàn thêm về chuyên đề này.Những lời giải cho các bài bất đẳng thức được sử dụng bởi những bất đẳng thức cổ điểnluôn mang một vẻ đẹp khó tả Tuy nhiên, có rất nhiều bài khi sử dụng các kết quả cổ điểnthì bị hạn chế Ví dụ như bài toán sau:

Biến đổi vế trái như sau:xkyk(xn+yn

) = (1 − ε)k(1 + ε)k[(1− ε)n+ (1 + ε)n]= (1 − ε2)k

1− nε +n(n − 1)2 ε2+ + 1+nε +n(n − 1)2 + = (1− ε2)k

2+ (n − 1)nε2+ ε4( )xkyk(xn+yn) g (1 − kε2 ε

2+ (n − 1)nε2+ 4( )= ε4( )( 1− kε2 )( )

) + (1 − kε2 2+ (n − 1 nε4) ( )∗g 2 − 2kε2

+ (n − 1)nε2

− k(n − 1)nε4

Trang 3

Trang 5

GIẢI TÍCH HÀM NHIỀU BIẾNBÁO CÁO

Tức là ta cần 2 g 2−2kε2

+ (n − 1)nε2− k(n − 1)nε4 Biến đổi một chút, ta thu đượck(n− 1)nε2 n

g ( − 1)n − 2k.Cho ε → 0 thì (n− 1 n − 2k) f 0 ô gk (n − 1)n2 Đến đây, ta sẽ kết luận được k =(n − 1)n

2 là giá trị cần tìm bằng cách chứng minh bấtđẳng thức (xy)(n−1)n2 (xn+ yn) f 2 đúng với điều kiện đề bài Bất đẳng thức này ta có thểchứng minh được bằng quy nạp.

Rõ ràng, việc xử lý bài toán trên thì bất đẳng thức cổ điển tỏ ra không mấy hiệu quả (vì bấtđẳng thức cổ điển thường đánh giá dựa vào điểm rơi, trong khi ví dụ trên điểm rơi khônggiúp ích được nhiều) Lúc này ta mới có một ý tưởng là xét một lân cận đủ nhỏ quanhđiểm rơi (ở (∗) để có đánh giá tiếp theo, ta cần 1−kε2

g 0, điều này hoàn toàn có thể doviệc chọn ε) sau đó là cho bán kính lân cận tiến về 0 Nói đơn giản, ta đã xấp xß điểm rơithay vì là trực tiếp làm bằng điểm rơi.

Ví dụ trên là điển hình về ưu thế của các phương pháp giải tích trong giải bất đẳng thức:cho phép ta giải bất đẳng thức bằng cách làm việc với đẳng thức (hoặc gần với đẳng thứckhi xấp xß đủ tốt) Nhận thấy được ưu điểm này, nhóm người viết muốn đề cập đến mộtphương pháp để giải bất đẳng thức, đó là khảo sát hàm số và nổi bật làphương phápnhân tử Lagrange Đây là một phương pháp có nhiều ưu điểm, cho phép làm việc với

hầu hết là đẳng thức để giải bất đẳng thức.

2 Nội dung

Nhắc lại lý thuyết

Điều kiện cần - Định lý nhân tử Lagrange:

Giả sử các đạo hàm riêng của f tồn tại, f đạt cực trị tại x0với ràng buộc ϕ(x) = 0 và∇ϕ(x) = 0Rn Khi đó tồn tại số thực λ sao cho:

∇ f (x0) = λ ∇ϕ(x0).

Điều kiện đủ: Cho U mở trong Rn, f : U → R, ϕ : U → R thỏa f,ϕ ∈ C2(U) Giả sử(x0,λ0) là điểm dừng của hàm Lagrange L(x, λ ) và ∇ϕ(x0) = 0 Xét dạng toàn phươngA u( ) =

n∑i, j=1

∂2L(x, λ0)∂ xixj

Trang 6

Giá trị lớn nhất, giá trị nhỏ nhất (cực trị toàn cục)

Cho U mở, bị chặn trong Rnvà f : U → R Giả sử biên của tập U (ký hiệu là ∂U, đượcđịnh nghĩa bởi ∂U = U\U) xác định bởi mặt cong ϕ(x) = 0, tức là

∂U = {x ∈ Rn:ϕ(x) = 0}.Ta biết rằng f đạt giá trị lớn nhất trên U Giả sử

f(x0) = max{ (x),x ∈ U}.fKhi đó, xảy ra một trong hai trường hợp sau đây:

•Nếu x0∈ U thì f đạt cực trị không điều kiện tại x0.•Nếu x0∈ ∂U thì f đạt cực trị với ràng buộc ϕ(x) = 0 tại x0

Như vậy hàm f chß có thể đạt giá trị lớn nhất tại các điểm cực trị không điều kiện hoặccực trị với ràng buộc ϕ(x) = 0 trên U Do đó, để tìm giá trị lớn nhất và nhỏ nhất của hàmftrên U, ta chß cần so sánh giá trị của hàm f tại các điểm dừng của f và điểm dừng củahàm Lagrange L trên U

Như vậy, ta có thể sử dụngĐịnh lý nhân tử Lagrange để tìm các điểm dừng của hàm đa

biến f với ràng buộc ϕ(x) = 0 Với mục đích tìm giá trị lớn nhất và nhỏ nhất trong chủđề này, việc xét đến điều kiện đủ của định lý là không quá cần thiết Nếu như trong cựctrị toàn cục, miền xác định là một miền mở, bị chặn thì trong các bài toán ta xét, miềnxác định là một "đường", tức là một miền không mở, và cũng bị chặn (bởi các yếu tố nhưa b, ,c > 0hay là rút ra từ chính điều kiện đề bài) Như vậy, ta rút ra được các bước chínhđể giải (hoặc định hướng cách giải) một bài toán tìm giá trị lớn nhất hay nhỏ nhất của mộthàm đa biến với điều kiện ràng buộc như sau:

Bước 1: Thiết lập hàm Lagrange L(x,λ) = f (x) −λϕ( )x.

Bước 2: Thiết lập và giải hệ phương trình tọa độ điểm dừng:

∂ L∂ x1

= ∂ L∂ x2

= =∂ L∂ xn

=∂ L∂ λ = 0.Từ đó tìm được các điểm dừng của f với ràng buộc ϕ(x) = 0.

Bước 3: Xét giá trị lớn nhất (hay nhỏ nhất) của hàm f tại các biên của miền xác định (nếu

hàm không có biên thì thác triển thành một hàm mới F bao gồm hàm f và bao đóng Ccủa nó).

Bước 4: So sánh các giá trị của f tại các điểm dừng và tại các giá trị biên (hoặc cận biên).

Nếu giá trị lớn nhất (hay nhỏ nhất) đạt tại biên của F nhưng không là biên của f thì kếtluận f không đạt giá trị lớn nhất (hay nhỏ nhất) Các trường hợp còn lại ta đều kết luậnđược giá trị lớn nhất và nhỏ nhất của hàm f

Mở rộng vấn đề: Điều kiện ràng buộc là nhiều phương trình.

Xét bài toán: tìm giá trị lớn nhất và nhỏ nhất (nếu có) của hàm số f liên tục trên Rnvới n

Trang 5

Trang 7

GIẢI TÍCH HÀM NHIỀU BIẾNBÁO CÁO

biến x1,x2, ,xnvà m phương trình ràng buộc (m < n) là m hàm số liên tụcg g1, 2, ,gm.Tương tự với trường hợp 1 phương trình ràng buộc, xét hàm số Lagrange:

L= f + λ1(b1−g1) + λm(b1−gm)(hàm n + m biến số).

Điều kiện cần (tương tự trường hợp có 1 phương trình ràng buộc): Tồn tại m số thựcλ1, ,λm(nhân tử Lagrange) sao cho(x1, ,xn,λ1, ,λm)là một điểm dừng của LNhư vậy, để tìm tọa độ điểm dừng của hàm Lagrange ta phải giải hệ phương trình gồmm+ n ẩn số.

Trường hợp 3 biến, 2 phương trình: Hàm f (x,y,z) với điều kiệng1(x, y, z) = 0g2(x, y, z) = 0.Giải hệ phương trình:∂ L

∂ x=∂ L∂ y=∂ L

∂ z= ∂ L∂ λ1 = ∂ L

Ví dụ về ứng dụng phương pháp nhân tử Lagrange

Trong không gian cho trước tam giác ABC và một điểm D thay đổi sao cho ABCDlà một tứ diện có thể tích V không đổi Tìm vị trí điểm D để tổng diện tích các mặtbên của tứ diện ABCD là nhỏ nhất.

Lời giải

Gọi H là hình chiếu của D lên mặt phẳng chứa tam giác ABC.Đặt AB = c, BC = a, AC = b, DH = h thì , ,c,h không đổi.a b

Gọi h h ha, b, clần lượt là khoảng cách từ D lên các đường thẳng BC CA, ,ABvà da,db,dc

lần lượt là khoảng cách từ H đến các đường thẳng BC CA AB, ,

Nhận xét: Hình chiếu của điểm D cần tìm lên mặt phẳng (ABC) phải nằm ở miền trong

của tam giác ABC Thật vậy, ta có:

h2+ d2

h2+ d2,SDCA=12bhb=1

h2+ d2.Xét một điểm M có hình chiếu lên mặt phẳng (ABC) là N sao cho MN = h và N nằmngoài tam giác ABC Không mất tổng quát, ta có hai trường hợp có thể xảy ra của vị trí

Trang 8

điểm N: điểm N nằm trong góc đối đßnh của BAC(hình bên trái) và điểm N nằm tronggóc BAC(hình bên phải) Với mỗi trường hợp, gọi E, F lần lượt là hình chiếu củaNlênAB, AC và nếu điểm N′như trong hai hình cùng các hình chiếu E′, F′từ N′đến AB, AC(các trường hợp N thuộc đường thẳng AB hoặc AC là các trường hợp suy biến, hoàn toàntương tự).

Với điểm N cho trước và điểm N′ được chọn như trong hình, ta có da′<da, db′<

db, dc′<dc(với da′, db′, dc′lần lượt là khoảng cách từ N′tới BC CA AB, , ), suy raSDAB′<S

Tóm lại, hình chiếu của điểm cần tìm lên mặt phẳngD (ABC) phải nằm ở miền trong củatam giác ABC.

Do đó ta được ada+bdb+cdc= 2SHBC+ 2SHCA+ 2SHAB= 2SABC= k không đổi.Tổng diện tích các mặt bên của tứ diện ABCD là:

2 a

h2+ d2+ b

h2+ d2+ ch2+ d2

h2+ x2+b hh2+y2+c

2+ z2.

Cách 1: Phương pháp nhân tử Lagrange

Xét hàm Lagrange:

L(x, y, z, λ ) =a

h2+ x2+ b

h2+y2+ch2+ z2

− λ (ax + by + cz − k).

Trang 7

Trang 9

GIẢI TÍCH HÀM NHIỀU BIẾNBÁO CÁO

Tọa độ điểm dừng là nghiệm của hệ phương trình:

∂ L∂ x= 0∂ L∂ y= 0∂ L∂ z= 0∂ L∂ λ= 0

h2+ x2− λ a = 0by

h2+ y2− λ b = 0cz

h2+ z2− λ c = 0ax+ by + cz=k

x= λh2+ x2

y= λh2+ y2

z= λh2+ c2

ax+ by + cz=k

1− λ2"x2= λ2

1− λ2"y2= λ2h2

"1− λ2"

z2= λ2

h2ax+ by + cz=kx, y, z,λ > 0, 1 − λ2

= 0⇐⇒

x= y = z =√λ h1− λ2

1− λ2= kh(a + b + c)

x= y = z = ka+ + cbλ = k

a+ + cb

Cách 2: Sử dụng bất đẳng thức cổ điển

Áp dụng bất đẳng thức Minkowski:

(ah)2+ (ax)2+

(bh)2+ (by)2+

( )ch2+ (cz)2

g12(ah+bh+ ch)2+ (ax + by + cz)2=12

(ha+hb+ hc)2+ k2.

Đẳng thức xảy ra khix

ax+ by + cz = k⇐⇒ x = y = z =a+ b + ck Vậy giá trị lớn nhất của S đạt được khi và chß khi x = y = z = k

a+ + cb

Trở về bài toán ban đầu: Ta đã tìm được giá trị nhỏ nhất của tổng diện tích các mặt bên

là S =12

(ha + hb + hc)2+ k2khi H cách đều ba cạnh của tam giác ABC, hay nói cáchkhác H là tâm đường tròn nội tiếp tam giác ABC, suy ra D nằm trên trục đi qua tâm nộitiếp tam giác ABC và vuông góc với (ABC).

Bình luận: Cách 2 ngắn gọn hơn nhiều so với cách 1 vì chß áp dụng trực tiếp bất đẳng

thức Minkowski mà không cần phải qua một bước trung gian nào cả Tuy nhiên, cái bấtlợi ở đây cũng bắt nguồn từ sự "ngắn gọn" đó bởi do yêu cầu người làm bài phải biết đến

Trang 10

bất đẳng thức Minkowski và áp dụng cụ thể vào bài này với những bộ số nào Cách 1 tuydài dòng hơn, nhưng người làm có thể không cần nhớ bất cứ một kết quả nào về bất đẳngthức cổ điển hay kỹ thuật chọn điểm rơi mà chß cần biết đạo hàm và có kỹ năng giải hệphương trình Phương pháp này rất phù hợp cho những ai bị hạn chế về kiến thức và kỹnăng bất đẳng thức, không cần biết quá nhiều về các kết quả của bất đẳng thức cổ điểnvẫn có thể giải bất đẳng thức được!

∂ L∂ a=10a

b −6b

a3 − λ 14a13= 0∂ L

∂ b=−5a2

b2 +9b

a2 − λ 14b13= 0∂ L

∂ λ= a14+ b14

− 2 = 0Do a,b > 0 nên từ hai phương trình đầu ta có:

10ab −6b

14a13 =−5a2

− 9k7+ 10k2

− 6 trên ( + )0, ∞ Ta chứng minh h(k) tăng ngặt.Thật vậy, ta có:

h′(k) = 45k8 k− 63k6+ 20

= k(k− 1)2(45k5+ 90k4+ 72k3+ 54k2+ 36k + 18) + 2k > ,0 ∀k > 0.Do đó h tăng trên (0,+∞) Vìh(1) = 0nên

5k9− 9k7+10k2− 6= ⇐⇒0 h(k) = h 1)( ⇐⇒k= 1.

Trang 9

Trang 11

GIẢI TÍCH HÀM NHIỀU BIẾNBÁO CÁO

Suy ra a = b, kết hợp với a14+b14= 2 thu được(a, b) = (1, 1).

Vậy hàm số có một điểm dừng là (1 1, ) Kiểm tra được (1 1, ) là cực tiểu của f nênf(a,b) g f (1,1) = , ∀( , ) ∈ R8 a b 2 Hoàn tất chứng minh.

Bình luận: Đối với những bất đẳng thức mà điều kiện dạng đa thức với số mũ lớn thì việc

sử dụng bất đẳng thức cổ điển là không dễ dàng Như ở ví dụ này, ta thấy cách cổ diển thểhiện sự bất lực khi phải xử lý số mũ 14 (bất đẳng thức cổ điển thường làm giảm số mũkhi đánh giá trội thay vì làm tăng) Trong khi đó phương pháp Lagange lại tỏ ra hiệu quảvì không cần xử lý quá nhiều về số mũ 14 mà hệ tọa độ điểm dừng cũng khá dễ dàng giảiđược bằng cách đưa về phương trình một ẩn.

Bất đẳng thức thứ nhất có thể thu được bằng việc sử dụngBất đẳng thức Cauchy cùng

vài đánh giá đại số Ở đây ta sẽ chứng minh bất đẳng thức thứ hai.

Xét hàm Lagrange: L(a,b,c,λ) = ab + bc + ca −abc−λ(a2+ b2+ c2 )+ abc − 4Tọa độ điểm dừng là nghiệm của hệ phương trình:

∂ L∂ a= 0∂ L∂ b= 0∂ L∂ c= 0∂ L∂ λ= 0

b+ c − bc − λ (2a + bc) = 0a+ c − ac − λ (2b + ac) = 0b+ − ab − λ (2c + ab) = 0aa2+ b2+ c2

+ abc − 4 = 0⇐⇒

λ =b+ c − bc2a+ bcλ =a+ c − ac

2b+ acλ =b+ a − ab

2c+ aba2+ b2+c2+ abc = 4

λ =b+ c − bc2a+ bc =

a+ c − ac2b+ ac =

b+ − aba2c+ aba2+ b2+ c2+ abc = 4

Từ đó ta có:

b+ c − bc2a+ bc =

a+ c − ac2b+ aca+ c − ac

2b+ ac =

b+ − aba2c+ aba2+ b2+c2+ abc = 4

Trang 12

Trường hợp 1:Có ít nhất 2 số bằng nhau (không mất tính tổng quát, giả sử a = b), thayvào (*) ta được:

a= bb+ c − bc

2 + bc =2b+ b2

2c+ b2 (1)2b2+ c2+ b2c= 4 ( )2Từ (1) ta thu được: (c + 2)(b2+ c − ) = 0 ⇐⇒ c = 2 − b2 2.

Thay c = 2 −b2vào (2) ta được:(b− 1 b − 2 b + 2 2b)( )( )( 2 − b − 2) = 0

Giải phương trình trên, kết hợp điều kiện a,b,c dương suy ra tọa độ các điểm dừng là(a b, ,c) = (1, 1,1),1 +√17

Trường hợp 2:3 số đôi một khác nhau:

(∗) ⇐⇒

2(b + a) + c − 2c(b + a) − c2 2=02(b + c) + 2a− a2 (b + ) −c a2=0a2+ b2+c2+ abc = 4

Trừ vế theo vế hai phương trình đầu, ta được: (a − c)(a + 2b + 2c) = 0, điều này là vô lývì a = c và a,b,c > 0 Vậy không tồn tại a b, ,cthỏa mãn trường hợp 2.

Bằng tính toán trực tiếp và so sánh các giá trị của hàm tại các điểm dừng và tại biên, ta cóf(a, b, c) f f (1,1 1, ) = 2 với mọi a b, ,cthỏa mãn điều kiện đề bài Hoàn tất chứng minh.

Bình luận: Đạo hàm của hàm đa thức vẫn là hàm đa thức nên việc biến đổi đại số khá dễ

dàng và phương trình cũng tương đối dễ giải Từ đó cũng thấy được rằng phương pháp nàyrất dễ sử dụng cho hàm dạng đa thức.

L(a b, ,c,λ1,λ2) = f (a, b,c) − λ1g a( , , c) − λb 2h a( , , c)b= a2b+ b2c+ c2a− λ1(a + b + c) − λ2

a2+b2+c2− 6".

Trang 11

Trang 13

GIẢI TÍCH HÀM NHIỀU BIẾNBÁO CÁO

Điểm dừng của L là nghiệm của hệ phương trình:

2ab+ c2

− λ1− 2λ2a= 02bc+ a2

− λ1− 2λ2b= 02ac+ b2

− λ1− 2λ2c= 0a+ + c = 0ba2+b2+ c2= 6Cộng vế theo vế 3 phương trình đầu: (a + +b c)2

− 3λ1− 2λ2(a + b + c) = 0 ⇒ λ1= 0.Cũng từ 3 phương trình đầu ta suy ra (quy ước mẫu bằng 0 thì tử cũng bằng 0):

a"2ab+ c2"

+ b"2bc+ a2"

4a2b2+4b2c2+4a2c2+ 4abc2+4bca2+4acb2+a4+b4+c4"= 6"

a2+b2+c2"2+ +2(ab + bc ca)2

= 636 + 2

(a+ + cb )2

−"a2+ b2+ c2"2

= 6(36 + 2 9 ) = 324.Ta suy ra được

a2b+b2c+c2af 6.

Ta đi tìm giá trị (a,b,c) khi đẳng thức xảy ra Đẳng thức xảy ra khi

a2b+b2c+ c2a= 6

Từ (1) suy ra

2abc+ a3= ab2 2+ bc2

2abc+ b3= bc2 2+ ca2

2abc+ c3= ca2 2+ ab2

Trang 14

Cộng vế theo vế ta được: 6abc + a3+ b3+c3= 3"ab2+bc2+ca2" ( )4Ta có: (a+ b + c)3= a3+ b3+ c3+ 3(a + b b + c c + a)( )( )

⇐⇒ a3+b3+c3

= −3"a2b+b2c+ c2a"

− 3"ab2+bc2+ca2"− 6abc.Thay vào (4) ta được:

ab2+ bc2+ ca2"+"

a2b+ b2c+ c2a"= 0⇐⇒ 2"ab2+bc2+ca2"

+ 2"

a2b+b2c+ c2a"="

a2b+b2c+ c2a"= 6⇐⇒ 2"ab2+bc2+ca2+a2b+b2c+ c2a+ 3abc"

− 6abc=6⇐⇒ 2(a + b + c)(ab + bc +ca) = abc + 6.6

Mà a + b + c = 0 nên abc = −1 (5) Từ (2), (3) ta tính được ab + bc + ca = −3 (6) Từ(2), (5), (6) suy ra a,b,c là 3 nghiệm của phương trìnhx3− 3x+ 1 =0.

Đặt x = 2cost, t ∈ [0,π], ta được phương trình:

8 cos3t−6 cost+ 1 = 0 ⇐⇒ cos3t = −12⇐⇒

Vậy nên ta kết luận giá trị lớn nhất của P là 6 khi a = 2cos2π

9,b= 2 cos4π

9 ,c= 2 cos8π9.

Bình luận: Đây là một ví dụ về việc sử dụng phương pháp nhân tử Lagrange để tìm giá

trị điểm rơi, phục vụ cho chứng minh bất đẳng thức Ta có thể thấy được ở trên, điểmrơi không hề dễ dàng nghĩ ra được Tới cả khi lập được hệ tọa độ điểm dừng thì việcgiải hệ đó cũng không đơn giản Vậy nên ta sẽ linh hoạt, sử dụng đồng thời yếu tố giảitích và bất đẳng thức cổ điển Rõ ràng, nhờ phương pháp Lagrange mà ta tìm được điểmrơi của bất đẳng thức, sau đó dựa vào điểm rơi để vận dụng bất đẳng thức Cauchy-Schwarz.

Trang 13

Trang 15

GIẢI TÍCH HÀM NHIỀU BIẾNBÁO CÁO

b2+ k(a2+ b2) g2ab+ 2.

Những bài toán như thế này rất hay khi bàn về vấn đề tìm các hằng số tốt nhất cho một bấtđẳng thức Đây là một vấn đề hấp dẫn cho những người ra đề hoặc là những người thíchtìm hiểu sâu về chuyên đề này Cách làm thường gặp là sẽ thay một số giá trị cụ thể củabiến vào bất đẳng thức để được một bất đẳng thức theo tham số, thông thường sẽ thay giátrị điểm rơi của bất đẳng thức Tuy nhiên, ở bài toán này, điểm rơi dễ thấy nhất là(1, 1),khi thay vào ta luôn được đẳng thức nên không thu được đánh giá nào của k Lúc này, ta sẽđổi về bài toán tìm giá trị nhỏ nhất (hoặc lớn nhất) của một hàm số nhiều biến, và phươngpháp nhân tử Larange có thể giúp ích rất nhiều trong việc đó Ta có lời giải với phươngpháp nhân tử Lagrange như sau:

Nhận xét:Ta luôn thu được đẳng thức khi thay a b= = 1 nên ta chß cần xét các trường hợp(a, b) = (1,1 , hay a = b Biến đổi bất đẳng thức đề bài như sau:)

b2+ k(a2

+ b2

) g2kab+ 2⇐⇒a12+1

b2−ab2 g k(2ab− a2

a, b > 0a+ b = 2

ab(2 − 4ab + 2a2b2) =

4(1 − ab)

2ab(1 − ab)2= 2ab(1 − )ab .Đặt g a( , b) = a+ − 2 Thiết lập hàm Lagrange:b

L(a b, ,λ ) = f (a,b) − λ ( , ) =g ab ab(1 − )2 ab − λ (a b+ − 2).

Ngày đăng: 02/08/2024, 16:20

w